Difference between revisions of "1988 AHSME Problems/Problem 26"

(Solution)
m (Solution)
Line 16: Line 16:
 
==Solution==
 
==Solution==
 
We can rewrite the equation as <math>\frac{\log{p}}{\log{9}} = \frac{\log{q}}{\log{12}} = \frac{\log{(p + q)}}{\log{16}}</math>. Then, the system can be split into 3 pairs: <math>\frac{\log{p}}{\log{9}} = \frac{\log{q}}{\log{12}}</math>, <math>\frac{\log{q}}{\log{12}} = \frac{\log{(p + q)}}{\log{16}}</math>, and <math>\frac{\log{p}}{\log{9}} = \frac{\log{(p + q)}}{\log{16}}</math>. Cross-multiplying in the first two, we obtain: <cmath>(\log{12})(\log{p}) = (2\log{3})(\log{q})</cmath> and <cmath>(\log{12})(\log{(p + q)}) = (2\log{4})(\log{q})</cmath>
 
We can rewrite the equation as <math>\frac{\log{p}}{\log{9}} = \frac{\log{q}}{\log{12}} = \frac{\log{(p + q)}}{\log{16}}</math>. Then, the system can be split into 3 pairs: <math>\frac{\log{p}}{\log{9}} = \frac{\log{q}}{\log{12}}</math>, <math>\frac{\log{q}}{\log{12}} = \frac{\log{(p + q)}}{\log{16}}</math>, and <math>\frac{\log{p}}{\log{9}} = \frac{\log{(p + q)}}{\log{16}}</math>. Cross-multiplying in the first two, we obtain: <cmath>(\log{12})(\log{p}) = (2\log{3})(\log{q})</cmath> and <cmath>(\log{12})(\log{(p + q)}) = (2\log{4})(\log{q})</cmath>
Adding these equations results in: <cmath>(\log{12})(\log{p(p+q)}) = (2\log{12})(\log{q})</cmath> which simplifies to <cmath>p(p + q) = q^2</cmath> Dividing by <math>pq</math> on both sides gives: <math>\frac{p+q}{q} = \frac{q}{p} = \frac{p}{q} + 1</math>. We set the desired value, <math>q/p</math> to <math>x</math> and substitute it into our equation: <math>\frac{1}{x} + 1 = x \implies x^2 - x - 1 = 0</math> which is solved to get our answer: <math>\boxed{\text{(C) } \frac{1 + \sqrt{5}}{2}}</math>.
+
Adding these equations results in: <cmath>(\log{12})(\log{p(p+q)}) = (2\log{12})(\log{q})</cmath> which simplifies to <cmath>p(p + q) = q^2</cmath> Dividing by <math>pq</math> on both sides gives: <math>\frac{p+q}{q} = \frac{q}{p} = \frac{p}{q} + 1</math>. We set the desired value, <math>q/p</math> to <math>x</math> and substitute it into our equation: <math>\frac{1}{x} + 1 = x \implies x^2 - x - 1 = 0</math> which is solved to get our answer: <math>\boxed{\text{(C) } \frac{1 + \sqrt{5}}{2}}</math>. -lucasxia01
  
 
== See also ==
 
== See also ==

Revision as of 00:28, 8 June 2016

Problem

Suppose that $p$ and $q$ are positive numbers for which

$\log_{9}(p) = \log_{12}(q) = \log_{16}(p+q)$

What is the value of $\frac{q}{p}$?

$\textbf{(A)}\ \frac{4}{3}\qquad \textbf{(B)}\ \frac{1+\sqrt{3}}{2}\qquad \textbf{(C)}\ \frac{8}{5}\qquad \textbf{(D)}\ \frac{1+\sqrt{5}}{2}\qquad \textbf{(E)}\ \frac{16}{9}$


Solution

We can rewrite the equation as $\frac{\log{p}}{\log{9}} = \frac{\log{q}}{\log{12}} = \frac{\log{(p + q)}}{\log{16}}$. Then, the system can be split into 3 pairs: $\frac{\log{p}}{\log{9}} = \frac{\log{q}}{\log{12}}$, $\frac{\log{q}}{\log{12}} = \frac{\log{(p + q)}}{\log{16}}$, and $\frac{\log{p}}{\log{9}} = \frac{\log{(p + q)}}{\log{16}}$. Cross-multiplying in the first two, we obtain: \[(\log{12})(\log{p}) = (2\log{3})(\log{q})\] and \[(\log{12})(\log{(p + q)}) = (2\log{4})(\log{q})\] Adding these equations results in: \[(\log{12})(\log{p(p+q)}) = (2\log{12})(\log{q})\] which simplifies to \[p(p + q) = q^2\] Dividing by $pq$ on both sides gives: $\frac{p+q}{q} = \frac{q}{p} = \frac{p}{q} + 1$. We set the desired value, $q/p$ to $x$ and substitute it into our equation: $\frac{1}{x} + 1 = x \implies x^2 - x - 1 = 0$ which is solved to get our answer: $\boxed{\text{(C) } \frac{1 + \sqrt{5}}{2}}$. -lucasxia01

See also

1988 AHSME (ProblemsAnswer KeyResources)
Preceded by
Problem 25
Followed by
Problem 27
1 2 3 4 5 6 7 8 9 10 11 12 13 14 15 16 17 18 19 20 21 22 23 24 25 26 27 28 29 30
All AHSME Problems and Solutions

The problems on this page are copyrighted by the Mathematical Association of America's American Mathematics Competitions. AMC logo.png